Recent content by saadhusayn

  1. saadhusayn

    I I want to expand a Gaussian wavepacket in terms of sines

    Yes, you're right about the upper limit being ##1##. Also, the fact that ##\sigma << 1## implies that we might as well take the upper limit to ##+\infty##. But other than a factor in front, I don't see how that changes my answer.
  2. saadhusayn

    I I want to expand a Gaussian wavepacket in terms of sines

    From this paper, I am trying to compute the coefficients in the expansion of the Gaussian wavepacket $$\phi(x) = \frac{1}{(2\pi\sigma^2)^{\frac{1}{4}}}\exp \Big(-\frac{(x-x_{0})^{2}}{4\sigma^{2}} + ik_{0}(x-x_{0})\Big) $$ where $$\sigma << 1$$and $$k_{0} >> \frac{1}{\sigma}$$ in terms of the...
  3. saadhusayn

    A Two loop Feynman diagram with quartic vertex

    I am trying to calculate the effective potential of two D0 branes scattering in Matrix theory and verify the coefficients in this paper: K. Becker and M. Becker, "A two-loop test of M(atrix) theory", Nucl. Phys. B 506 (1997) 48-60, arXiv:hep-th/9705091. The fields are expanded about a constant...
  4. saadhusayn

    Calculating effective action at two loops

    Does this mean that the expression for the above vertex is $$ -\frac{g}{2}\epsilon^{abx}\epsilon^{cdx}\int d\tau \langle A_{a} (\tau) A_{c} (\tau)\rangle \langle Y^{i}_{b} (\tau)Y^{i}_{d}(\tau) \rangle $$
  5. saadhusayn

    A Calculating the ghost field in the Becker and Becker paper

    This is the paper that I refer to. I'm trying to figure out the ghost action (Equation 2.16) in the background field gauge. I am attempting to use Srednicki's (chapter 78) expression for the ghost field in the background gauge. However, I am missing out on a √g coefficient in front of the term...
  6. saadhusayn

    Expanding an action about a background field

    If we expand the action ##S[\phi]## about ##\varphi(x)##, ## S[\phi] = S[\varphi] + \int d^d x \frac{\delta S[\varphi]}{\delta \varphi (x)} f(x) + \frac{1}{2!}\int d^dx_1 d^d x_2 \frac{\delta^2 S[\varphi]}{\delta \varphi(x_1) \delta \varphi(x_2)} f(x_1) f(x_2) + \mathcal{O}(f^3)## But...
  7. saadhusayn

    Perturbation expansion with path integrals

    I expanded the exponential with the derivative to get: ## Z = \Bigg(1 + \frac{1}{2} \frac{\partial}{\partial x_{i}} A^{-1}_{ij} \frac{\partial}{\partial x_{j}} + \frac{1}{4} \frac{\partial}{\partial x_{i}} A^{-1}_{ij} \frac{\partial}{\partial x_{j}} \frac{\partial}{\partial x_{k}} A^{-1}_{kl}...
  8. saadhusayn

    Finding the Ricci tensor for the Schwarzschild metric

    I'm trying to use Cartan's method to find the Schwarzschild metric components from Hughston and Tod's book 'An Introduction to General Relativity' (pages 89-90). I'm having problems calculating the components of the Ricci tensor. The given distance element is $$ ds^2 = e^{2 \lambda} dt^2 -...
  9. saadhusayn

    Divergence of the energy momentum tensor

    I need to prove that in a vacuum, the energy-momentum tensor is divergenceless, i.e. $$ \partial_{\mu} T^{\mu \nu} = 0$$ where $$ T^{\mu \nu} = \frac{1}{\mu_{0}}\Big[F^{\alpha \mu} F^{\nu}_{\alpha} - \frac{1}{4}\eta^{\mu \nu}F^{\alpha \beta}F_{\alpha \beta}\Big]$$ Here ##F_{\alpha...
  10. saadhusayn

    I Matrix for transforming vector components under rotation

    In the original frame, \mathbf{A} = \begin{pmatrix} 1 \\ 0 \\...
  11. saadhusayn

    I Matrix for transforming vector components under rotation

    Say we have a matrix L that maps vector components from an unprimed basis to a rotated primed basis according to the rule x'_{i} = L_{ij} x_{j}. x'_i is the ith component in the primed basis and x_{j} the j th component in the original unprimed basis. Now x'_{i} = \overline{e}'_i. \overline{x} =...
  12. saadhusayn

    I Transformation of covariant vector components

    Riley Hobson and Bence define covariant and contravariant bases in the following fashion for a position vector $$\textbf{r}(u_1, u_2, u_3)$$: $$\textbf{e}_i = \frac{\partial \textbf{r}}{\partial u^{i}} $$ And $$ \textbf{e}^i = \nabla u^{i} $$ In the primed...
  13. saadhusayn

    A Confusion regarding the $\partial_{\mu}$ operator

    The ##\mu## is contravariant and the ##\beta ## covariant, right? Shouldn't $$ \partial_{x}\partial^{x}$$ and $$ \partial_{t}\partial^{t}$$ have opposite signs, since we are working with four vectors?
  14. saadhusayn

    A Confusion regarding the $\partial_{\mu}$ operator

    Thank you. Is this correct? $$ \mathcal{L} =\frac{1}{2}g^{\mu \alpha} (\partial_{\mu} \phi)(\partial_{\alpha} \phi) - \frac{1}{2}m^{2} \phi^{2}$$ So $$\frac{\partial \mathcal{L}}{\partial (\partial_{\beta} \phi)} = \frac{1}{2}g^{\mu\alpha}(\partial_{\mu} \phi...
  15. saadhusayn

    A Confusion regarding the $\partial_{\mu}$ operator

    $$ \frac{1}{2}(\partial_{\mu} \phi)^2 = \frac{1}{2}(\partial_{\mu} \phi)(\partial^{\mu} \phi) $$
Back
Top